Endocrine, Diabetes & Metabolism, MALE REPRODUCTIVE SYSTEM, Female Reproductive System & Breast (2), Female Reproductive System & Breast (1), Cardiovascular system (2), Cardiovascular system (1), social science
A 50-year-old man undergoes an abdominal CT scan for left flank pain and is diagnosed with a kidney stone. The CT also shows a 2-cm left adrenal mass. The lesion is homogenous with well-defined margins and low xray attenuation. His flank pain is resolved after passing the stone. He has no other symptoms. He takes valsartan for mild hypertension. Family history is unremarkable. His blood pressure is 132/85 mm Hg and pulse is 67/min. His BMI is 25 kg/m . The physical examination is unremarkable. Basic metabolic panel is within normal limits. What is the most appropriate next step in management of this patient? A. Evaluation for hormonal secretion B. Left adrenalectomy C. MRI of the abdomen D. No further workup E. Repeat CT scan in 6 months
Correct answer A Educational objective: Adrenal incidentalomas should be evaluated for malignant features and biochemical hyperactivity. Initial testing should include overnight dexamethasone test and urinary fractionated catecholamines and metanephrines. Hypertensive patients also should have measurement of aldosterone and plasma renin activity to exclude hyperaldosteronism.
A 26-year-old nulligravid woman comes to the office for evaluation of infertility. She has been trying to conceive for the past 18 months with her partner of 3 years. He has a child from a previous relationship. Her menses started at age 12 and cycles are irregular; she bleeds for 10 days every 2-3 months. The patient has been sexually active since age 16 with 3 male partners and has never used contraception other than condoms. She does not use tobacco, alcohol, or illicit drugs. Blood pressure is 112/78 mm Hg and pulse is 82/min. BMI is 23 kg/m . The patient has mild acne and some hair growth on her upper lip and in the mid-abdominal region. Which of the following management options is most likely to lead to pregnancy? A. Basal body temperature monitoring B. Clomiphene citrate C. Semen analysis D. Spironolactone E. Weight loss
Correct answer B Educational objective: Polycystic ovary syndrome (PCOS) is characterized by menstrual irregularity and ovulatory dysfunction, sometimes resulting in infertility. Weight loss and clomiphene citrate are the first-line treatments for infertility due to PCOS.
A 59-year-old man with ischemic cardiomyopathy comes to the physician with painful enlargement of both breasts for the last 2-3 weeks. He also has significant fatigue, loss of libido, and erectile dysfunction. The patient's medications include candesartan, carvedilol, spironolactone, furosemide, atorvastatin, and aspirin. Bilateral tender gynecomastia is seen on examination. Lungs are clear to auscultation. Testicular examination is unremarkable. Which of the following is the most appropriate next step in management of this patient? A. Discontinue candesartan and start enalapril B. Discontinue carvedilol and start metoprolol succinate C. Discontinue spironolactone and start eplerenone D. Obtain serum prolactin and TSH levels E. Obtain serum testosterone levels
Correct answer C Educational objective: Aldosterone receptor blockers - spironolactone and eplerenone - improve survival in systolic heart failure with low left ventricular ejection fraction. Both medications may cause hyperkalemia. Spironolactone has significant anti-androgenic effects. Eplerenone has fewer endocrine side effects.
According to the American Diabetes Association, hemoglobin A1c can be used to diagnose diabetes mellitus. Which of the following does the Association recommend as the threshold for diagnosing diabetes? A. 5% B. 5.5% C. 6% D. 6.5% E. 7%
Correct answer D Educational objective: Current American Diabetes Association guidelines recommend using hemoglobin A1c >6.5% for diagnosing diabetes. Hb A1c 5.7%-6.4% is associated with high risk for future diabetes; Hb A1c <5.7% is considered normal.
3. A 16-year-old boy is brought to the emergency department with abdominal and groin pain that began suddenly 10 hours ago. The pain started in the lower abdomen and then radiated to the right groin. He has vomited 4 times. The boy is sexually active with 2 female partners and does not use condoms. His temperature is 37.2 C (99 F), blood pressure is 104/62 mm Hg, and pulse is 108/min. He has mild lower abdominal tenderness with palpation. The right hemiscrotum is swollen, erythematous, and tender to palpation. The right testicle is firm with a horizontal lie with no cremasteric reflex. Which of the following is the most important next step in management of this patient? A. Emergency surgery B. Empiric antibiotics for gonorrhea and chlamydia C. Scrotal support and analgesia D. Transillumination of the scrotum E. Ultrasound of the scrotum with Doppler
ANSWER A Educational objective: Testicular torsion is a urologic emergency that presents with acute, severe scrotal pain, horizontal testicular lie, and an absent cremasteric reflex. Ultrasound is not required for the diagnosis of classic cases. Definitive management consists of surgical detorsion and fixation with exploration of the contralateral hemiscrotum and should be done within 6 hours to maximize viability.
5. A 65-year-old man comes to the urgent care clinic complaining of frequent urination and perineal pain for the last 2 days. He also mentions malaise and chills. He has never had similar symptoms before. He is in a monogamous relationship with his wife. The patient's temperature is 37.8 C (100 F), blood pressure is 120/70 mm Hg, pulse is 94/min, and respirations are 18/min. Rectal examination shows a boggy and tender prostate. Laboratory results are as follows: Hemoglobin 13.1 g/dL White blood cells 12,000/µL with 7% band forms Platelets 329,000/µL The patient's creatinine level is 0.9 mg/dL. Dipstick urinalysis is positive for leukocyte esterase. Which of the following is the most appropriate next step in management of this patient? A.. Alpha-adrenergic blocking agents B. CT scan of the pelvis C. Culture of mid-stream urine sample D. Prostate specific antigen measurement E. Urethral catheterization
ANSWER C Educational objective: Acute bacterial prostatitis presents in a manner similar to other urinary tract infections but with the addition of perineal pain, pronounced systemic symptoms (fever, chills, acute illness), and a tender, boggy prostate on examination. Urine culture should be obtained to help direct antibiotic therapy.
4. A 63-year-old man comes to the urgent care clinic with scrotal pain over the last 2 days. He has also been experiencing urinary frequency and urgency attributed to a "prostate problem." His temperature is 38.1 C (100.8 F), blood pressure is 130/75 mm Hg, pulse is 86/min, and respirations are 15/min. Physical examination shows left scrotal swelling and a tender scrotal mass; there is no urethral discharge. Laboratory studies show a white blood cell count of 14,000/µL with a left shift. Urinalysis shows bacteriuria and pyuria. Which of the following is the most likely organism responsible for this patient's findings? A. Chlamydia trachomatis B. Escherichia coli C. Neisseria gonorrhoeae D. Pseudomonas E. Staphylococcus aureus
Answer B Educational objective: Acute epididymitis in younger patients is usually due to sexually transmitted organisms such as Chlamydia trachomatis or Neisseria gonorrhoeae. Empiric therapy should include ceftriaxone and doxycycline. In older men, epididymitis is usually non-sexually transmitted and caused by common gram-negative organisms. Fluoroquinolones are effective in most cases.
7. A 66-year-old man comes to the office due to 3 months of scrotal discomfort. He reports heaviness and a dull ache in the left scrotal area that is worse toward the end of the day. He has no fever, dysuria, hematuria, or penile discharge. The patient has a history of hypertension and takes amlodipine. He had a vasectomy 20 years ago. He lives with his wife and has 2 grown children. On examination, the abdomen is soft and nontender without organomegaly. Examination in the standing position shows a palpable, soft, irregular, nontender mass above the left testis that disappears in the supine position. There is no skin redness, groin bulge, or transillumination. Both testes are normal in size and without tenderness. Which of the following is the most appropriate next step in management of this patient? A. Annual scrotal ultrasound B. CT scan of abdomen and pelvis C. Scrotal support and analgesics D. Serial semen analysis E. Surgical referral
Answer C Educational objective: Varicoceles are due to tortuous dilation of the pampiniform plexus surrounding the spermatic cord. Asymptomatic patients do not require treatment; those with scrotal discomfort usually improve with analgesics (eg, nonsteroidal anti-inflammatory drugs) and/or scrotal support.
1. A 63-year-old man reports nocturia once or twice a night, and he has noticed a decrease in the size and force of his urinary stream over the past 6 months. On examination, he appears healthy. General physical examination is within normal limits. On digital rectal examination, his prostate is mildly enlarged with a 1.5cm hard nodule in the right lateral lobe of the prostate. His prostate-specific antigen level is 1.5 ng/mL (normal <4 ng/mL). Which of the following is the best next step in this patient's management? A. Order MRI B. Reassessment of symptoms in 6 months C. Repeat prostate-specific antigen level in 6 months D. Schedule transrectal ultrasound-guided prostate biopsy E. Start tamsulosin and dutasteride
Answer D Educational objective: Prostatic nodules found on digital rectal examination should be evaluated promptly by transrectal ultrasound-guided prostatic biopsy, regardless of the prostate-specific antigen level.
6. A 27-year-old man comes to the office due to a scrotal lump that his girlfriend noticed prior to sexual intercourse. The patient has no testicular pain or associated symptoms. Medical history is unremarkable, and the patient takes no medications. He does not smoke and drinks 2 or 3 alcoholic beverages a day. Vital signs are normal. Examination shows a 2-cm, nontender mass in the left testicle. Transillumination reveals no appreciable transmission of light. Which of the following is the most appropriate next step in management of this patient's condition? A. Antibiotic therapy B. Beta-hCG and alpha fetoprotein assay C. CT scan of the abdomen and pelvis D. MRI of the pelvis and scrotum E. Scrotal ultrasound
Answer E Educational objective: Testicular cancer is the most common solid tumor in young men, and typically presents with a painless testicular mass. Scrotal ultrasound is the initial diagnostic test of choice as it can quickly differentiate intrinsic from extrinsic lesions and determine if testicular cancer is likely.
2. A 16-year-old boy is brought to the office for evaluation of "heaviness" in his left hemiscrotum. Initial examination in the standing position shows normal external male genitalia. When the patient performs a Valsalva maneuver, a soft, irregular, nontender mass is palpated in the left hemiscrotum. The left hemiscrotum does not transilluminate. These findings disappear when the patient is supine. Which of the following is the most likely diagnosis in this patient? A. Hydrocele B. Inguinal hernia C. Spermatocele D. Testicular cancer E. Varicocele
Answer E Educational objective: Varicoceles are caused by dilation of the veins of the pampiniform plexus and may present with scrotal heaviness. The classic finding is a palpable, soft, irregular mass on scrotal examination that worsens with the Valsalva maneuver.
A 45-year-old woman develops abdominal pain and vomiting in the recovery room after uncomplicated elective cholecystectomy. Her medical history is significant for systemic lupus erythematosus and type 1 diabetes mellitus. Her outpatient medications include insulin, prednisone, and hydroxychloroquine. The patient's temperature is 38 C (100.4 F), blood pressure is 72/38 mm Hg, and pulse is 110/min and regular. She has a round, plethoric face and central obesity. Fingerstick glucose is 66 mg/dL. High-flow intravenous fluids are initiated, but she remains hypotensive. Which of the following is the most appropriate management for this patient? A. Dexamethasone B. Dobutamine C. Glucagon D. Vancomycin E. Vasopressin
Correct answer A Educational objective: Acute adrenal insufficiency is a potentially lethal postoperative complication that may be seen in patients on chronic glucocorticoid therapy. Features include nausea, vomiting, abdominal pain, hypoglycemia, and hypotension. Management includes intravenous glucocorticoids and high-flow intravenous fluids.
A 23-year-old woman comes to the office due to palpitations, increased frequency of bowel movements, and a 4.5-kg (10-lb) weight loss despite adequate food intake. The patient's symptoms also include redness of the eyes associated with tearing, grittiness, and photophobia, as well as intermittent diplopia when reading or watching television. Medical history is unremarkable; she has never been pregnant. The patient is not sexually active. She has smoked half a pack of cigarettes a day for the last 2 years. Blood pressure is 138/66 mm Hg and pulse is 110/min and regular. The patient appears anxious. The thyroid gland shows diffuse symmetrical enlargement without tenderness. A thyroid bruit is audible. Ocular examination shows bilateral proptosis with conjunctival injection and periorbital edema. Laboratory evaluation shows a serum TSH level of 0.005 μU/mL and a free thyroxine level of 4.1 ng/dL (normal: 0.9-2.4). Thyroid scintigraphy shows 24-hour radioactive iodine uptake of 56% (normal: 10%-30%) with a diffuse pattern. The patient is started on a beta blocker and oral prednisone. Which of the following additional interventions should be initiated at this time? A. Methimazole B. Propylthi
Correct answer A Educational objective: Antithyroid drugs (eg, methimazole) can induce remission in patients with mild Graves disease. They are also used to attain a euthyroid state in preparation for definitive treatment with radioactive iodine or thyroidectomy in patients with moderate to severe disease. Radioactive iodine can worsen Graves ophthalmopathy and should be avoided in patients with significant ocular symptoms.
A 24-year-old woman with type 1 diabetes mellitus has 6 months of increasing fatigue and occasional lightheadedness. The patient takes 2-3 naps during the day due to feeling exhausted. Her blood pressure is 92/60 mm Hg, and pulse is 95/min. The patient's body mass index is 18 kg/m . Physical examination shows hyperpigmentation around her lips and in the palmar creases. Her serum creatinine and liver function tests are normal. Which of the following is the most likely cause of this patient's symptoms? A. Adrenal insufficiency B. Dilated cardiomyopathy C. Hemochromatosis D. Hypothyroidism E. Insulin resistance
Correct answer A Educational objective: Chronic primary adrenal insufficiency can present with chronic fatigue, generalized weakness, weight loss, gastrointestinal symptoms, postural hypotension, and hyperpigmentation or vitiligo. Testing should include early morning measurement of serum cortisol and ACTH and a high-dose ACTH stimulation test.
A 4-day-old infant is in the nursery with poor feeding and intermittent vomiting. The infant was born vaginally after an otherwise uncomplicated pregnancy. Gender has not been assigned due to ambiguous genitalia. Temperature is 36.7 C (98 F), blood pressure is 49/18 mm Hg, pulse is 192/min, and respirations are 42/min. The patient's pulse oximetry shows 98% on room air. Examination shows slow capillary refill and mottled extremities. There is clitoromegaly, partially fused labia, and bilaterally non-palpable gonads. The rest of the examination is normal. Intravenous access is established and resuscitation with normal saline is initiated. Which of the following is the most appropriate next step in the management of this infant? A. Measure serum 17-α hydroxyprogesterone level B. Measure serum testosterone level C. Measure serum TSH level D. Perform abdominal ultrasound E. Perform echocardiography
Correct answer A Educational objective: Classic "salt-wasting" congenital adrenal hyperplasia (CAH) causes virilization in females and hypotensive shock. All infants with ambiguous genitalia should be screened for CAH by 17-α hydroxyprogesterone testing. Emergency treatment of adrenal crisis consists of volume expansion and glucocorticoids.
A 6-day-old boy is brought to the emergency department with a 2-day history of decreased oral intake and fussiness and a 1-day history of vomiting. According to his parents, "he wouldn't wake up to feed this afternoon." He is exclusively breastfed and has been making a normal number of wet diapers. The infant was born vaginally to a 23-year-old primigravid mother following an uncomplicated pregnancy. Review of systems is negative for fever or sick contacts. His birth weight was 3.5 kg (7.7 lb); current weight is 3 kg (6.6 lb). His temperature is 36.7 C (98 F), blood pressure is 70/30 mm Hg, pulse is 205/min, and respirations are 48/min. Physical examination shows a lethargic infant with a sunken fontanelle, poor skin turgor, and a capillary refill time of >5 seconds. The extremities are cool. No hepatosplenomegaly, petechiae, or purpura is seen. He has a normal uncircumcised phallus and bilaterally descended testicles. An intravenous line is placed and 20 mL/kg of normal saline is administered. Laboratory results are as follows Complete blood count Hemoglobin 14 g/dL Hematocrit 44% Platelets 240,000/ µL Leukocytes 13,000/ µL Serum chemistry Sodium 122 mEq/L Potassium 6.4 mEq/L chloride 9
Correct answer A Educational objective: Congenital adrenal hyperplasia (CAH) classically presents with hyponatremia, hyperkalemia, hypoglycemia, and hypotension, as well as virilization in girls. Although CAH is included in newborn screening, infants with severe 21-hydroxylase deficiency can present before results are available. The diagnosis is based on elevated 17-hydroxyprogesterone levels.
A 72-year-old woman is hospitalized for a severe urinary tract infection complicated by vomiting, altered mentation, and hypotension and is admitted to the intensive care unit for intravenous fluids and antibiotics. On the second hospital day, the patient develops atrial fibrillation with rapid ventricular response. Medical history is significant for hypertension. Results of thyroid function studies are as follows: TSH 0.14 µU/mL Free T4 1.1 ng/dL (normal: 0.9-2.4) Total T3 60 ng/dL (normal: 70-195) Which of the following is the most likely cause of this patient's abnormal thyroid studies? A. Euthyroid sick syndrome B. Graves disease C. Pituitary apoplexy D. Silent thyroiditis E. Toxic multinodular goiter
Correct answer A Educational objective: Euthyroid sick syndrome (ESS) is a constellation of abnormal thyroid function tests seen in severe illness. Serum T3 is low, TSH and T4 may be normal or low, and reverse T3 is elevated. ESS usually resolves spontaneously and does not require treatment unless the abnormalities persist after the patient has returned to baseline health.
A 28-year-old woman comes to the office with her 32-year-old spouse for an infertility evaluation. Neither has any children, and they have been trying to conceive for over a year without success. Both are healthy. The patient reports regular 28-day menstrual cycles without molimina symptoms. Physical examination is normal. Her spouse recently had a normal semen analysis. Which of following is the best next step in management of this patient? A. Day 21 progesterone level B. Endometrial biopsy C. Male partner karyotype D. Mycoplasma cultures E. Postcoital test
Correct answer A Educational objective: Infertility is evaluated with a semen analysis in men and tests for ovulatory function (eg, day 21 progesterone level), ovarian reserve, and tubal patency in women. An elevated day 21 progesterone level is suggestive of recent ovulation.
A 49-year-old man with type 2 diabetes mellitus comes to the office for follow-up. He is currently taking the maximum daily dose of metformin. The patient has been counseled about his diet and exercise but has had a 3.2-kg (7-lb) weight gain over the last year. His BMI is 33 kg/m . Serum creatinine is 0.9 mg/dL, and hemoglobin A1c is 7.8%. Which of the following is the most appropriate second medication for improving this patient's glycemic status? A. Exenatide B. Glimepiride C. Insulin glargine D. Pioglitazone E. Sitagliptin
Correct answer A Educational objective: Injectable glucagon-like peptide-1 (GLP-1) analogues (eg, exenatide, liraglutide) are useful in patients with type 2 diabetes who fail metformin monotherapy. GLP-1 analogues are associated with weight loss and low hypoglycemic risk
A 61-year-old postmenopausal woman comes to the office due to leakage of urine. The patient reports increased urinary frequency and voids 1 or 2 times per hour. She often awakens at night with an immediate need to void and occasionally does not make it to the bathroom in time. The patient has mild intermittent asthma controlled with an albuterol inhaler. Her other medications include low-dose aspirin and a multivitamin. She had 2 cesarean deliveries, the last of which was complicated by a venous thromboembolism after delivery. Vital signs are normal. BMI is 27 kg/m . Pelvic examination reveals a thin, atrophic vagina. Cough stress test is negative and postvoid residual volume is 65 mL. Urinalysis and urine culture are normal. Which of the following is the best next step in management of this patient? A. Bladder training with timed voids B. Midurethral sling procedure C. Pessary placement D. Transdermal estrogen patch E. Urodynamic testing
Correct answer A Educational objective: Lifestyle modifications and behavioral therapy (eg, pelvic muscle exercises, bladder training) are first-line treatments for urgency incontinence. Keeping a voiding diary, scheduling bladder emptying, and slowly increasing the time between voids can be an effective therapy for urgency incontinence. If first-line treatments fail, pharmacologic therapy (eg, antimuscarinics, beta-3 agonists) is initiated.
A 43-year-old woman comes to the office due to feeling "down" and fatigued. The patient has gained weight over the last several months. Appetite and sleep are intact. She separated from her husband 6 months ago. The patient has a history of suicide attempts and bipolar disorder and has been treated with lithium for the past 3 years. On physical examination, there is psychomotor slowing but no ataxia or tremor. Muscle weakness is not present, but the relaxation phase of deep tendon reflexes is prolonged. Complete blood count and serum chemistries are normal; TSH concentration is 37 μU/mL. Serum lithium level is 1.0 mEq/L (range: 0.8-1.2). Which of the following is the most appropriate management for this patient? A. Continue lithium and prescribe levothyroxine B. Discontinue lithium and prescribe levothyroxine C. Discontinue lithium and prescribe sertraline D. Discontinue lithium and repeat thyroid function tests in 3 months E. Reduce lithium dose
Correct answer A Educational objective: Lithium can cause a number of thyroid abnormalities, especially goiter and hypothyroidism. Thyroid function should be assessed prior to and periodically during lithium therapy. Lithium-induced hypothyroidism may be treated with levothyroxine while continuing lithium therapy.
A 66-year-old woman is seen in the outpatient clinic for follow-up. The patient has no current symptoms. She has a history of hypothyroidism and takes levothyroxine replacement therapy. The patient's other medical problems include hypertension and acid reflux disease. She underwent dual-energy x-ray absorptiometry 2 months ago, which was normal. Her TSH level is <0.01 mU/L. Despite a long discussion, the patient refuses to decrease the dose of levothyroxine as lower doses make her feel "exhausted and sleepy all the time." Which of the following is a possible complication of levothyroxine overreplacement in this patient? A. Cardiac arrhythmia B. Dementia C. Hyperlipidemia D. Ophthalmopathy E. Thyroid cancer
Correct answer A Educational objective: Overtreatment of hypothyroidism with levothyroxine is associated with an increased risk of excessive bone loss and atrial fibrillation.
A 27-year-old woman comes to the physician due to poor sleep, hand tremors, and heat intolerance for the last 2-3 weeks. She has no fever or neck pain. Examination shows mild, diffuse thyroid enlargement. Her hands are warm and sweaty with a fine tremor. The remainder of the examination is unremarkable. Laboratory results show TSH <0.03 µU/mL and serum thyroxine (T ) of 20.0 µg/dL. Radioactive iodine uptake at 24 hours is 2% (normal 10%-30%). Which of the following is the best next step in management of this patient? A. Beta blocker B. High-dose glucocorticoid C. Methimazole D. Nonsteroidal anti-inflammatory drug E. Radioactive iodine ablation
Correct answer A Educational objective: Painless thyroiditis is characterized by a brief period of hyperthyroid symptoms followed by a hypothyroid phase and subsequent recovery. Patients with mild symptoms usually require no therapy; beta blockers are used for symptomatic thyrotoxic patients.
A 28-year-old woman, gravida 1 para 1, comes to the office due to ongoing nipple discharge despite stopping breastfeeding 2 months ago. She had an uncomplicated vaginal delivery 13 months ago and her child is healthy. Her last menstrual period was prior to her pregnancy. She is monogamous and sexually active with her husband and does not use contraception. Milky discharge is easily expressed from both breasts. No other abnormalities are seen. Urine β-hCG is negative. Laboratory results are as follows: Serum creatinine 0.7 mg/dL FSH 4 mIU/mL LH 3 mIU/mL Prolactin 209 ng/mL TSH 4.8 µU/mL Which of the following is the most appropriate next step in management of this patient?A. Brain MRIB. Breast ultrasoundC. Medroxyprogesterone acetate challengeD. Reassurance and follow-up in 1 monthE. Total T4, free T4, and free T3 levels
Correct answer A Educational objective: Pituitary prolactinomas present with galactorrhea, amenorrhea, and marked hyperprolactinemia (>200 ng/mL). Diagnosis is confirmed via brain MRI. First-line treatment is with dopamine agonists (eg, cabergoline, bromocriptine).
A 52-year-old woman with mild hypertension treated with losartan is found to have a serum calcium level of 10.8 mg/dL. She is asymptomatic. Repeat testing shows serum calcium of 11.1 mg/dL and creatinine of 0.8 mg/dL. Her parathyroid hormone (PTH) level is 100 pg/mL (normal 10-65 pg/mL), and urinary calcium is 300 mg/24 hours. TSH and 25-hydroxyvitamin D levels are normal. Which of the following is the most appropriate next step in evaluating this patient? A. Bone mineral density testing B. Bone-specific alkaline phosphatase C. Parathyroid hormone-related peptide (PTHrP) D. Renal ultrasound E. Ultrasound of the neck
Correct answer A Educational objective: Primary hyperparathyroidism is characterized by elevated calcium with elevated parathyroid hormone. Patients should be evaluated for indications for parathyroidectomy including: Age <50 Low bone density with a T-score <-2.5 (at hip, distal radius, or lumbar spine) Renal insufficiency Calcium >1 mg/dL above the upper limit of normal
A 78-year-old woman is brought to the physician by her daughter due to intermittent dizziness, weakness, and confusion. The patient was seen for fever and dysuria 3 days ago and started on trimethoprim/sulfamethoxazole. Her other medical problems include type 2 diabetes mellitus, hypertension, and chronic kidney disease with a baseline serum creatinine of 1.3 mg/dL. Her chronic medications include hydrochlorothiazide, amlodipine, glyburide, atorvastatin, calcium, vitamin D, and alendronate. Her temperature is 36.8 C (98.2 F), blood pressure is 140/85 mm Hg supine and 144/89 mm Hg standing, pulse is 84/min, and respirations are 20/min. BMI is 19 kg/m . There are no focal neurologic findings. Her Mini-Mental State Examination score is 24/30. Laboratory results are as follows Leukocytes 7,300/m Serum chemistry Sodium 136 mEq/L Potassium 3.6 mEq/L Bicarbonate 24 mEq/L Creatinine 1.4 mg/dL Glucose 70 mg/d Urinalysis Protein trace Leukocyte esterase trace Nitrites negative Bacteria few White blood cells 10-15/hpf Red blood cells 3-5/hpCasts noneHemoglobin A 6.1%Which of the following is the best next step in management of this patient?A. Discontinue glyburide B. Discontinue hydrochlorothiazide
Correct answer A Educational objective: Sulfonylureas (eg, glyburide) can accumulate in elderly patients with chronic kidney disease (CKD) and cause hypoglycemia; this effect is potentiated by trimethoprim/sulfamethoxazole. Goals for diabetes control should be relaxed in elderly individuals with CKD.
A 42-year-old man comes to the office after his blood pressure was found to be 146/92 mm Hg at a health fair. The patient has had no unusual headaches, blurry vision, polyuria, chest pain, or shortness of breath. He has no significant prior medical conditions and takes no medications. HIV testing 3 years ago was negative. The patient's father was diagnosed with colon cancer at age 65, and his brother underwent a thyroidectomy for differentiated papillary thyroid cancer at age 34. The patient has a current 15-pack-year smoking history and does not drink alcohol or use illicit drugs. Blood pressure is 140/90 mm Hg, and pulse is 68/min and regular. BMI is 27 kg/m . Repeat blood pressure measurement after 5 minutes is 140/92 mm Hg. Physical examination is unremarkable. Nonfasting laboratory results are as follows: Sodium 142 mEq/L Potassium 4.5 mEq/L Chloride 104 mEq/L Bicarbonate 24 mEq/L Blood urea nitrogen 11 mg/dL Creatinine 0.9 mg/dL Calcium 9.2 mg/dL Glucose 140 mg/dL Urine microalbumin/creatinine ratio, TSH, and ECG are normal. A fasting lipid profile has been ordered. Which of the following additional tests is recommended at this time? A. Hemoglobin A1c B. Low-dose CT scan of the che
Correct answer A Educational objective: The prevalence of type 2 diabetes mellitus is increased in patients with hypertension, and patients with both hypertension and diabetes are at increased risk for cardiovascular complications. Patients with hypertension should be screened for diabetes with a fasting glucose level, hemoglobin A1c, or oral glucose tolerance test.
A 17-year-old girl is evaluated for vulvar pruritus and vaginal discharge that began yesterday. She recently had a urinary tract infection that was treated with antibiotics. The patient is sexually active with a male partner and has pain with intercourse. She is afebrile and vital signs are normal. Pelvic examination reveals vulvar and vaginal erythema and a scant white vaginal discharge with a pH of 4. Which of the following is the most likely etiology of this patient's symptoms? A. Candida albicans B. Chlamydia trachomatis C. Gardnerella vaginalis D. Mycoplasma genitalium E. Trichomonas vaginalis
Correct answer A Educational objective: Vulvovaginal candidiasis causes inflammation of the vulva and vagina, resulting in erythema and pruritus. It can be distinguished from other causes of vaginitis by a normal vaginal pH of 3.8-4.5.
A 37-year-old woman comes to the clinic for a routine health maintenance examination. She feels well and has no concerns. The patient has no chronic medical conditions but has not had an examination in >5 years. She takes no daily medications and does not use tobacco, alcohol, or illicit drugs. Family history is significant for a grandmother with hypothyroidism and an uncle who had an ischemic stroke at age 68. The patient is sexually active with a female partner with whom she has been in a monogamous relationship for the last 10 years. Blood pressure is 120/70 mm Hg and pulse is 78/min. BMI is 24 kg/m . Which of the following screening tests are recommended at this visit? A. Cervical cancer screening B. Chlamydia and gonorrhea screenings C. Diabetes mellitus screening D. No screening indicated at this time E. Thyroid function screening
Correct answer A Educational objective: Women who have sex with women face multiple barriers to adequate health care and are often underscreened. Cervical cancer screening should be performed every 3-5 years in immunocompetent women age 21-65 regardless of sexual orientation.
A 23-year-old woman comes to the office due to several days of yellow vaginal discharge. She is sexually active with a male partner and uses oral contraceptives. A Pap test a year ago was normal. A urine pregnancy test is negative. The cervix is erythematous and swollen and bleeds when the cotton swab is inserted. There is no cervical motion or adnexal tenderness. Wet mount microscopy shows numerous white blood cells but no motile organisms. Which of the following is the best next step in management of this patient? A. Cervical histopathology and cytology B. Empiric ceftriaxone and doxycycline C. Empiric metronidazole D. Gram stain and culture of cervical fluid E. Transvaginal ultrasound
Correct answer B Educational objective: Acute cervicitis can present with postcoital bleeding, cervical friability, and a mucopurulent vaginal discharge. Acute cervicitis is a clinical diagnosis, and empiric treatment is with ceftriaxone and doxycycline.
A 32-year-old woman comes to the office due to palpitations and fatigue over the last 2 months. She delivered a healthy child 9 months ago. The patient gained excessive weight during her pregnancy and has been trying to lose it. Blood pressure is 140/80 mm Hg and pulse is 100/min. Her BMI is 32 kg/m . The thyroid gland is nontender and not enlarged. Laboratory results are as follows: TSH 0.01 µU/mL Triiodothyronine (T ) 340 ng/dL Free T 0.9 ng/dL Radioactive iodine uptake <1% (normal 10%-30%) Thyroglobulin 4 ng/mL (5-40 ng/mL) Antithyroglobulin antibodies negative Which of the following is the most likely diagnosis? A. Chronic autoimmune thyroiditis B. Factitious thyrotoxicosis C.Graves' disease D. Postpartum thyroiditis E. Subacute (de Quervain's) thyroiditis
Correct answer B Educational objective: Factitious thyrotoxicosis causes hyperthyroid symptoms associated with elevated T3, suppressed TSH, and low radioactive iodine uptake and thyroglobulin.
A 32-year-old woman is evaluated for heat intolerance and palpitations. She has no past medical problems. Nine months ago, she delivered a healthy boy via an uncomplicated vaginal delivery. A month ago, she was treated symptomatically for an upper respiratory infection. Blood pressure is 144/70 mm Hg, and pulse is 110/min. Examination shows no proptosis. The thyroid gland is diffusely enlarged without nodules or tenderness. Laboratory results are as follows: TSH 0.005 µU/mL Free T4 3.2 ng/dL (normal: 0.9-2.4) Serum T3 410 ng/dL (normal: 70-195) Radioactive iodine uptake at 24 hours is 32% (normal: 10%-30%) with a diffuse pattern. Which of the following is the most likely diagnosis? A. Euthyroid sick syndrome B. Graves disease C. Postpartum thyroiditis D. Silent thyroiditis E. Subacute thyroiditis
Correct answer B Educational objective: Graves disease is caused by antibodies to the TSH receptor. It typically presents with hyperthyroidism and a diffuse goiter. Ophthalmopathy is present at the time of diagnosis in about 25% of cases. Radioiodine uptake is increased with a diffuse pattern.
A 48-year-old woman comes to the office due to abnormal bleeding. The patient first noticed postcoital spotting 3 months ago. For the past month, the spotting has become increasingly heavy, requiring her to use a tampon or pad after intercourse, and has been associated with a malodorous vaginal discharge. Pelvic examination shows a firm, exophytic, friable, 1-cm lesion that extends laterally on the posterior aspect of the cervix. A thin, watery, malodorous discharge is noted throughout the vaginal vault. Which of the following is the most likely diagnosis in this patient? A. Acute cervicitis B. Cervical cancer C. Cervical ectropion D. Cervical polyp E. Prolapsed leiomyoma
Correct answer B Educational objective: Invasive cervical cancer can present with abnormal bleeding (eg, postcoital spotting); a malodorous vaginal discharge; and a friable, exophytic cervical lesion
A 56-year-old woman is evaluated for persistent fatigue. A year ago, she developed panhypopituitarism following resection of a large nonfunctional pituitary tumor. Current treatment includes hydrocortisone and levothyroxine, with her most recent dose adjustment a few months ago. The patient has had no associated headaches, excessive urination, visual disturbances, or weight change. Vital signs are normal. BMI is 26 kg/m with no change in weight over the last 6 months. Physical examination is unremarkable. Laboratory results are as follows: Sodium 137 mEq/LPotassium 3.9 mEq/L Chloride 104 mEq/L Bicarbonate 22 mEq/L Blood urea nitrogen 6 mg/dL Creatinine 0.8 mg/dL Calcium 9 mg/dL Glucose 76 mg/dL TSH 0.3 µU/mL An MRI of the pituitary 6 months ago showed no residual pituitary tumor. Which of the following is the most appropriate next step in management of this patient's disorder? A. Add liothyronine therapy B. Measure free T4 level C. Measure insulin-like growth factor-1 level D. Measure total T3 level E. Reduce levothyroxine dose
Correct answer B Educational objective: Levothyroxine therapy in central hypothyroidism should be based on serum free T4 levels, with the dose adjusted to maintain T4 in the high-normal range. TSH in central hypothyroidism demonstrates minimal responsiveness to levothyroxine and may not rise in patients with inadequate replacement.
A 76-year-old man comes to the physician due to fatigue and lethargy for several weeks. He has a non-ischemic cardiomyopathy with left ventricular ejection fraction of 25%, an automated implantable cardioverter-defibrillator, and a history of ventricular tachycardia that has been controlled with amiodarone over the past year. Neck examination is unremarkable. Laboratory evaluation shows TSH of 22 µU/mL and low T3/T4 levels. Which of the following is the best next step in management of this patient? A. Check anti-thyroid peroxidase antibodies B. Continue amiodarone and start levothyroxine C. Discontinue amiodarone and repeat thyroid function tests in a few weeks D. Discontinue amiodarone and start levothyroxine E. Order thyroid ultrasound
Correct answer B Educational objective: Long-term amiodarone use can cause thyroid dysfunction (hypothyroidism or hyperthyroidism). Patients with overt hypothyroidism may continue amiodarone but should initiate thyroid replacement therapy.
A 34-year-old woman comes to the physician due to fatigue. She has a 6-month history of progressive fatigue associated with weight gain, dry skin, and poor sleep. The patient's past medical history is unremarkable. She is married and has 2 children under age 3. On examination, she has no thyroid enlargement or tenderness. TSH is 14 µU/mL and serum free thyroxine (T ) is 0.5 ng/dL. In addition to initiating levothyroxine, which of the following is the most appropriate next step in management of this patient? A. Morning serum cortisol level B. No additional workup or therapy C. Radioactive iodine uptake D. Thyroid peroxidase antibody test E. Thyroid ultrasound
Correct answer B Educational objective: Most patients with primary hypothyroidism have chronic autoimmune (Hashimoto) thyroiditis. Patients with typical hypothyroid symptoms associated with an elevated TSH level and low free thyroxine can be treated with levothyroxine without additional diagnostic evaluation.
A 3-day-old full-term boy is brought to the office for his first newborn examination after an uncomplicated delivery and nursery course. The parents are concerned that the boy appears to have breasts. The patient has been breastfeeding well with normal urine and stool output. Pubertal changes started in both parents around age 12. A maternal aunt has hypothyroidism, but family history is otherwise unremarkable. Weight, length, and head circumference measurements are average. Vital signs are normal. Examination shows firm, palpable, disc-like tissues under both areolae, and a small amount of white, milky discharge is expressed from both nipples. External genitalia appear normal, and there is no pubic or axillary hair. Which of the following is the most appropriate next step in management of this patient? A. Gram stain and culture of discharge B. Reassurance and observation C. Serum FSH level D. Serum prolactin level E. Ultrasound of the breasts
Correct answer B Educational objective: Neonatal breast hypertrophy and galactorrhea are benign findings caused by stimulation from maternal hormones. Spontaneous regression occurs over several months, and reassurance should be provided.
A 38-year-old woman comes to the physician due to occasional headaches. She was diagnosed with hypertension several months earlier and treated with a number of different agents without significant improvement. Her blood pressure is 174/102 mm Hg and pulse is 82/min. Her BMI is 28 kg/m. Physical examination is unremarkable. Laboratory results are as follows:Hemoglobin: 14.4 g/dLCreatinine: 0.8 mg/dLPotassium: 3.1 mEq/LPlasma renin activity: UndetectablePlasma aldosterone concentration: 35 ng/dL (normal 7-30 ng/dL)Which of the following medications is most appropriate for controlling this patient's hypertension? A. ChlorthalidoneB. EplerenoneC. LabetalolD. NifedipineE. Telmisartan
Correct answer B Educational objective: Patients with hypertension and plasma aldosterone concentration to plasma renin activity ratio >20 have likely primary hyperaldosteronism. Surgery is indicated for primary hyperaldosteronism due to an adrenal adenoma; aldosterone antagonists (eg, spironolactone, eplerenone) are preferred for bilateral adrenal hyperplasia
A 65-year-old woman comes to the office to discuss the results of a screening DXA test. She has been healthy all her life, has no chronic medical conditions, and takes no medications. The patient experienced menopause at age 52; she was treated with combined estrogen/progestin menopausal hormone therapy due to hot flashes but discontinued the treatment after a few months due to concerns regarding adverse effects. She has experienced a 1 cm (0.4 in) loss of height since that time. The patient quit smoking 30 years ago and drinks wine only on social occasions. She lives alone and has no children. Her mother died of breast cancer at age 56 and her father died at age 82 from a stroke. BMI is 22 kg/m . Physical examination is unremarkable. DXA reveals a T score of −2.1 and −2.6 at femoral neck and lumbar spine, respectively. Complete blood count, comprehensive metabolic panel, and serum phosphorus are normal. Which of the following additional investigations should be performed for this patient at this time? A. 24-hour urine calcium excretion B. 25-hydroxyvitamin D level C. Lateral spine x-ray D. No additional investigation E. Serum protein electrophoresis
Correct answer B Educational objective: Patients with osteoporosis warrant laboratory testing for reversible secondary causes. The evaluation should include serum chemistries (eg, calcium, phosphorus, albumin, total protein, and renal and hepatic function markers) and complete blood count. In addition, vitamin D stores should be assessed with a 25- hydroxyvitamin D level.
A 23-year-old woman comes to the physician due to lethargy and palpitations during the first trimester of pregnancy. Thyroid function tests are ordered. Which of the following changes are expected during a normal pregnancy? A. Decreased TSH, decreased total T4 B. Decreased TSH, increased total T4 C. Increased TSH, decreased total T4 D. Increased TSH , increased total T4 E. No significant changes in TSH or total T4
Correct answer B Educational objective: Pregnancy induces a number of changes in thyroid physiology: increased serum thyroxine-binding globulin, increased thyroid hormone production, and higher total thyroid hormone levels. Patients with primary hypothyroidism will require an increase in levothyroxine dose during pregnancy.
A 38-year-old primigravida at 10 weeks gestation comes to the office for her first prenatal visit. She has had occasional nausea and vomiting but otherwise feels fine. The patient has hypothyroidism that has been managed with a stable dose of levothyroxine for the last 3 years. Vital signs are normal. TSH is 4.7 µU/mL and total T4 is 8.5 µg/dL. Which of the following is the best next step in management of this patient? A. Decrease the dose of levothyroxine and repeat thyroid function test in 4 weeks B. Increase the dose of levothyroxine and repeat thyroid function test in 4 weeks C. Measure free T4 to adjust the dose of levothyroxine D. Measure T3 levels to adjust the dose of levothyroxine E. Repeat thyroid function test in 4 weeks on the current dose of levothyroxine
Correct answer B Educational objective: Pregnant patients with hypothyroidism require an increased dose of levothyroxine as soon as they become pregnant due to their bodies' inability to increase thyroxine appropriately. The dose of levothyroxine is subsequently adjusted based on TSH levels using pregnancy-specific norms (or maintenance of TSH between the lower reference limit of normal and <2.5 µU/mL).
A 26-year-old woman comes to the office due to persistent fatigue and inability to lose weight. She has also experienced hair loss and dry skin. The patient was diagnosed with primary hypothyroidism 2 years ago and was started on levothyroxine therapy, which she takes on an empty stomach every morning. She exercises 5 times per week, follows very strict caloric restrictions, and has eliminated gluten-containing foods from her diet. Medical history is unremarkable, and the patient has never been pregnant. In addition to levothyroxine, she also takes a cyclic estrogen-progestin oral contraceptive. The patient is sexually active with one partner and does not use tobacco, alcohol, or illicit drugs. Blood pressure is 124/80 mm Hg and pulse is 70/min. BMI is 26 kg/m . The thyroid gland is diffusely enlarged, firm, and nontender, and the remainder of the physical examination is normal. Laboratory results are as follows: TSH 2.2 µU/mL Free T4 1.6 ng/dL (normal: 0.9-2.4) Which of the following is the most appropriate next step in management of this patient's condition? A. Add liothyronine (T3) to current therapy B. Continue current therapy and evaluate other etiologies C. Discontinue oral contra
Correct answer B Educational objective: Primary hypothyroidism is treated with levothyroxine, with the dose adjusted to maintain serum TSH in the normal (or low-normal) range. Further dose increases or the addition of liothyronine (T3) are not recommended. Hypothyroid symptoms are nonspecific, and patients with residual symptoms should be evaluated for alternate etiologies.
A 42-year-old woman comes to the emergency department complaining of headaches and vomiting. She has had panic attacks for the past couple of months that manifest as a racing heart, turning pale, and profuse sweating. At a recent clinic visit, she was started on propranolol due to elevated blood pressure and palpitations. The patient's current blood pressure is 240/140 mm Hg and heart rate is 88/min. On examination, her lungs are clear to auscultation. The abdomen is soft and non-distended with no palpable masses. There is no peripheral edema. There is 1+ protein and 1-3 red blood cells/high-power field in her urine. Laboratory results are as follows:Blood urea nitrogen 12 mg/dL Creatinine 1.2 mg/dL Thyroid-stimulating hormone 2.5 µU/mL Which of the following is the most likely cause of this patient's condition? A. Adrenal cortical disease B. Adrenal medullary disease C. Aortic coarctation D. Primary renal disease E. Renal vascular disease
Correct answer B Educational objective: Secondary causes of hypertension should be considered in patients who develop new-onset, severe hypertension that is refractory to medical therapy. Pheochromocytoma is characterized by episodic hypertension associated with sweating, headache, palpitations, and tachycardia.
A 52-year-old woman comes to the office due to a 3-day history of pain in the anterior lower neck radiating to the jaw. The pain is exacerbated by turning her head. The patient has also had palpitations and a feeling of being "hot." She experienced an upper respiratory infection 4 weeks ago, but otherwise her medical history is unremarkable. Temperature is 37.5 C (99.5 F), blood pressure is 144/80 mm Hg, pulse is 125/min and regular, and respirations are 18/min. Due to neck pain, the patient does not allow neck palpation. The oropharynx is not erythematous. Erythrocyte sedimentation rate is 100 mm/hr. TSH is 0.005 µU/mL and free thyroxine is 2.7 µg/dL (normal: 0.9- 2.4). Which of the following is the most likely diagnosis in this patient? A. Riedel thyroiditis B. Subacute thyroiditis C. Thyroid abscess D. Thyroid hemorrhage E. Thyroid lymphoma
Correct answer B Educational objective: Subacute thyroiditis is characterized by fever, neck pain, and a tender goiter, typically following a viral illness. Patients have a self-limited thyrotoxic phase followed by hypothyroidism and eventual recovery of thyroid function. Treatment is symptomatic with beta blockers and nonsteroidal anti-inflammatory drugs.
An 82-year-old woman comes to the office due to fatigue. For the past 4 months since her husband died, she has had low energy associated with decreased appetite. The patient has had no weight change, fever, chills, or night sweats. She sleeps 5-6 hours a night. Medical history is notable for hypertension, depression, and osteoporosis, for which the patient takes hydrochlorothiazide, sertraline, and alendronate, respectively. She does not smoke and drinks alcohol only once or twice a year on social occasions. Temperature is 36.8 C (98.2 F), blood pressure is 128/78 mm Hg, and pulse is 76/min. BMI is 24 kg/m . On examination, the patient is awake, alert, and cooperative. Neck examination is normal with no goiter. Cardiopulmonary examination is normal. Neurologic examination shows normal cranial nerves and motor strength. Ankle reflexes are absent. Laboratory studies show normal glucose, electrolytes, and blood counts. TSH is 7.2 µU/mL, and free T4 is 1.1 ng/dL (normal: 0.9-1.7). Which of the following is the most appropriate next step in management of this patient's symptoms? A. Initiate low-dose levothyroxine treatment B. Obtain periodic thyroid function studies C. Order serum free T3 lev
Correct answer B Educational objective: Subclinical hypothyroidism is defined as a serum TSH above the upper limit of normal with a serum free T4 level in the normal range. Patients age ≥70 with subclinical hypothyroidism should generally not be treated unless TSH is ≥10 µU/mL but should have periodic monitoring for progression to overt hypothyroidism.
A 20-year-old nulligravid woman comes to the office due to heavy menstrual periods. She has noticed heavier bleeding with regular cycles over the last year. Six months ago, she began taking combined oral contraceptives, but each month she continues to have heavy bleeding lasting up to 10 days. Hemoglobin is 9 g/dL. A transvaginal ultrasound reveals a 3-cm submucosal fibroid with a large intracavitary component that distorts the endometrial cavity. Which of the following is the most appropriate next step in management of this patient? A. Endometrial ablation B. Hysteroscopic myomectomy C. Levonorgestrel-releasing intrauterine device placement D. Switching to a different oral contraceptive E. Uterine artery embolization
Correct answer B Educational objective: Submucosal fibroids that cause abnormal uterine bleeding are managed with a hysteroscopic myomectomy in women who desire future fertility.
A 30-year-old woman without significant past medical history comes to the physician with a palpable lump in her neck. She has no associated symptoms and no family history of thyroid or other endocrine disorders. Her TSH level is 1.3 µU/mL (normal 0.5-5.0 µU/mL). Thyroid ultrasound shows a 16-mm round, hypoechoic nodule in the right lobe of the thyroid. There is no lymph node enlargement. Which of the following is the most appropriate course of action? A. Anti-thyroid peroxidase antibodies B. Fine-needle aspiration C. Repeat thyroid ultrasound in 6 months D. Right thyroid lobectomy E. Thyroid scintigraphy
Correct answer B Educational objective: TSH and thyroid ultrasound should be performed in all patients with thyroid nodules. The risk of malignancy can be assessed based on historical risk factors, TSH, nodule size, and ultrasound characteristics. High-risk nodules should be further evaluated with fine-needle aspiration.
A 13-year-old soccer player is brought to the clinic for follow-up on type 1 diabetes mellitus. He was diagnosed 1 month ago after presenting with polyuria, polydipsia, weight loss, and random blood glucose levels of 506 mg/dL and 496 mg/dL. He was started on a subcutaneous insulin regimen and initially had stable blood sugars ranging between 100-150 mg/dL. Over the past few weeks, his blood glucose levels have been consistently <90 mg/dL before each meal and at bedtime, and his insulin dose has decreased significantly. He is back in school but has not resumed sports yet. Which of the following is the most likely explanation for the changes in this patient's blood glucose levels? A. The patient is entering adolescence, an insulin-sensitive state, and his insulin requirement will continue to decrease B. The patient is in the remission phase of diabetes mellitus and his insulin requirement will likely increase again C. The patient is likely miscalculating the insulin dose and administering too much insulin D. The patient was incorrectly diagnosed with diabetes mellitus and can discontinue insulin therapy E. The patient's insulin requirement is decreasing because he is not exercising as muc
Correct answer B Educational objective: The "honeymoon" or "remission" phase of type 1 diabetes mellitus (T1DM) occurs several weeks after the diagnosis of new onset T1DM and is characterized by an increase in endogenous insulin production. It presents with decreased insulin requirement and can last for months to years.
A 36-year-old woman comes to the office for follow-up of hypothyroidism. She has no current symptoms. The patient was diagnosed with hypothyroidism secondary to Hashimoto's thyroiditis 4 years ago. She takes 150 µg levothyroxine daily before breakfast and her dose has been stable for the last year. Physical examination, including thyroid examination, is normal. Thyroid function tests are as follows: TSH 1.4 µU/mL (normal: 0.5-5.0 µU/mL) Free thyroxine (T ), serum 2.6 ng/dL (normal: 0.9-2.4 ng/dL) Triiodothyronine (T ), serum 65 ng/dL (normal: 70-195 ng/dL) Which of the following is the best next step in management of this patient? A. Add liothyronine (T ) to current therapy B. Continue current therapy C. Decrease levothyroxine dose D. Increase levothyroxine dose E. Obtain thyroid peroxidase antibody level
Correct answer B Educational objective: The therapeutic goal in primary hypothyroidism is to maintain TSH levels within the normal reference range by replacement with levothyroxine. Levothyroxine dose adjustment should be based on TSH levels alone rather than direct measurement of thyroid hormone levels.
A 64-year-old postmenopausal woman comes to the office due to 2 months of vulvar pruritus. The patient has severe itching, irritation, and soreness in the vulvar region, which interferes with her sleep. She has tried multiple moisturizers without relief. The patient has no vaginal discharge or dysuria. She takes metformin for type 2 diabetes mellitus and lisinopril for hypertension. She is married but has avoided sexual intercourse due to discomfort. Examination of the external genitalia is shown in the exhibit. Which of the following is the most likely diagnosis? A. Condyloma acuminata B. Lichen sclerosus C. Vulvar cancer D. Vulvovaginal atrophy E. Vulvovaginal candidiasis
Correct answer B Educational objective: Vulvar lichen sclerosus presents as intense pruritus of the vulvar region due to inflammation, followed by chronic scarring and atrophy. High-potency topical corticosteroids are the first-line treatment.
A 4-year-old girl is brought to the clinic due to 3 days of vaginal itching and dysuria. She was toilet trained at age 3 and has had no incontinence. Examination shows an overweight girl with mild vulvar erythema. There is a small amount of clear vaginal discharge and a crescentic hymen. No anal lesions are noted. Urinalysis is normal. Which of the following is the most likely cause of this patient's condition? A. Fungal overgrowth B. Helminthic infection C. Poor hygiene D. Sexually transmitted infection E. Vaginal voiding
Correct answer C Educational objective: Nonspecific vulvovaginitis presents in prepubertal girls (low estrogen) with vulvar pruritus, erythema, and vaginal discharge. Risk factors include poor toilet hygiene, scented soaps, and tight-fitting clothes. Treatment consists of avoidance of modifiable risk factors.
A 50-year-old woman comes to the office to follow up on type 2 diabetes mellitus. She was diagnosed 5 years ago and is treated with metformin at the maximal tolerated dose. The patient has been unable to lose weight despite following a healthy diet and exercising at least 5 times a week. She has no chronic complications of diabetes but has had recurrent episodes of vaginal candidiasis in the last 2 years. The patient works as a commercial bus driver for a tourist company and does not use tobacco, alcohol, or recreational drugs. Vital signs are normal. BMI is 27 kg/m . Physical examination is otherwise unremarkable. Serum creatinine is 1 mg/dL, fasting glucose is 156 mg/dL, and hemoglobin A1c is 7.9%. Six months ago, hemoglobin A1c was 7.2%. When added to metformin, which of the following medications is most appropriate for this patient? A. Basal insulin B. Dipeptidyl peptidase-4 inhibitor C. Glucagon-like peptide-1 receptor agonist D. Sodium-glucose cotransporter 2 inhibitor E. Sulfonylurea
Correct answer C Educational objective: Glucagon-like peptide-1 receptor agonists are very effective antidiabetes agents with the additional benefits of weight loss, low risk for hypoglycemia, and reduced risk for cardiovascular disease.
A 78-year-old woman is brought to the office for evaluation of a right heel wound. She had an ischemic stroke a year ago with residual right hemiplegia and has lived in a nursing home facility since. The patient also has oropharyngeal dysphagia and is fed through a gastrostomy tube. For the past several weeks, she has had a gradually enlarging right heel ulcer She has no pain sensation in the right leg and has had no fever or foulsmelling drainage. Her other medical problems include hypertension, type 2 diabetes mellitus, and coronary artery disease. Right heel examination shows a 2-cm ulcer with thick, firm eschar. There is no crepitus or surrounding redness. The lower-extremity pulses are weak. Which of the following is the best next step in management of this patient? A. Hyperbaric oxygen therapy B. Intravenous antibiotics C. Pressure redistribution with proper positioning D. Surgical debridement E. Wound swab culture
Correct answer C Educational objective: Heel pressure ulcers are usually managed conservatively as removing tissue around the heel can increase the risk of osteomyelitis. Proper positioning to redistribute the pressure is the most important intervention to improve healing
A 23-year-old woman comes to the office due to excess facial hair that she finds embarrassing. She has had this problem for years and has tried different methods of hair removal, with unsatisfactory results. The patient has no chronic medical conditions and has regular menstrual periods. She is not sexually active and takes no medications. BMI is 29 kg/m . On physical examination, there is coarse, dark hair on her upper lip, chin, midsternum, and buttocks. The rest of the examination is unremarkable. Which of the following is the best next step in management of this patient? A. Low-estrogen oral contraceptive pills B. Pelvic ultrasound C. Serum total testosterone test D. Spironolactone E. Thyroid function tests
Correct answer C Educational objective: Hirsutism is excess terminal hair growth in androgen-dependent areas (eg, chin, upper lip, upper abdomen, chest, back). All women with hirsutism require a serum total testosterone level to evaluate for possible underlying androgen disorders.
A 22-year-old man with no past medical history comes to the physician due to breast enlargement. The patient is 6' 2" (188 cm) tall and weighs 85 kg (187 lb). He has mild bilateral gynecomastia along with long extremities, sparse facial hair, and small, firm testes. Serum LH and FSH are elevated and testosterone is low. Serum prolactin and TSH are normal. Which of the following is most likely to establish the diagnosis in this patient? A. 17-hydroxyprogesterone level B. Genotyping for fibrillin-1 mutation C. Karyotype analysis D. MRI of the brain E. Testicular biopsy
Correct answer C Educational objective: Klinefelter syndrome is characterized by delayed puberty, tall stature, gynecomastia, small firm testes, and hypogonadism with elevated gonadotropins. Karyotype analysis confirms the diagnosis.
A 40-year-old obese man with type 2 diabetes mellitus comes to the clinic with abdominal distension, flatus, and low-volume fecal incontinence over the past 2 weeks. He has been treated with metformin for years and was recently started on exenatide and orlistat. His abdomen is distended, tympanic, and nontender. Which of the following is the best next step in management of this patient? A. Discontinue exenatide B. Lactose-free diet C. Low-fat diet D. Metoclopramide E. Simethicone
Correct answer C Educational objective: Orlistat inhibits pancreatic lipase to reduce fat absorption and increase fecal fat wasting. Patients with high dietary fat intake can develop side effects similar to fat malabsorption (eg, flatus, fecal incontinence, oily spotting).
A 66-year-old woman comes to the office to review the results of her recent DXA scan of the hips and spine. She has a history of essential hypertension, osteoarthritis, and acid reflux disease. Current medications include lisinopril, omeprazole, and acetaminophen as needed. She has never smoked and has no history of fractures or falls. Height is 165 cm (5 ft 5 in) and weight is 70 kg (154.3 lb). Laboratory testing, including renal function, electrolytes, TSH, and parathyroid hormone level, is normal. Results of the DXA scan are as follows: Bone density (g/cm ) Young adult (T score) Age-matched (Z score) Lumbar spine L1-4 (average) 0.916 −2.3 −0. Total proximal femur Left 0.641 −2.9 −1.8 Right 0.729 −2.2 −1.1 Mean 0.685 −2.6 −1.4 Which of the following is the most appropriate next step in management of this patient? A. Calculate FRAX score to decide management options B. Diagnose osteopenia and discuss management options C. Diagnose osteoporosis and discuss management options D. Obtain DXA scan of left forearm for further clarification E. Reassure the patient she has normal bone density based on Z scores
Correct answer C Educational objective: Osteopenia is defined as a T score of −1.1 to −2.4 and osteoporosis as a T score <−2.5. Diagnostic interpretation is based on the lowest T score at the lumbar spine (average bone density at L1-L4) and either hip.
A 51-year-old woman comes to the physician due to a persistent dry cough after an upper respiratory infection 2 months ago. She has no hemoptysis, chest pain, or weight loss. Chest examination is unremarkable. Laboratory results are as follows:Serum creatinine 0.7 mg/dL Serum calcium 11.0 mg/dL Intact parathyroid hormone 10 pg/mL (normal 10-60 pg/dL) 1,25-hydroxyvitamin D 106 ng/mL (normal 15- 80 ng/mL) Chest x-ray reveals right paratracheal and bilateral hilar adenopathy. CT reveals adenopathy and a 1.5- cm left lower lung nodule. Which of the following is the most likely cause of this patient's hypercalcemia? A. Multiple myeloma B. Parathyroid adenoma C. Sarcoidosis D. Squamous cell carcinoma E. Vitamin D intoxication
Correct answer C Educational objective: Patients with hypercalcemia and elevated 1,25-hydroxyvitamin D should have a chest x-ray to evaluate for granulomatous diseases (sarcoidosis) and lymphoma.
A 14-year-old boy is brought to the office for follow-up of newly diagnosed type 1 diabetes mellitus. He was diagnosed a week ago when he was admitted to the hospital with diabetic ketoacidosis. The patient was discharged on a treatment regimen including insulin glargine and insulin lispro. Since returning home, his fingerstick blood glucose levels have ranged from 140-180 mg/dL. The patient is referred for diabetes education. Which of the following is also advised for the patient at this time? A. Initiate insulin pump therapy B. Order complete blood count C. Order TSH level D. Order urine albumin-creatinine ratio E. Refer for dilated eye examination
Correct answer C Educational objective: Patients with type 1 diabetes mellitus are at increased risk for additional autoimmune disorders, especially autoimmune thyroid disease and celiac disease. Patients should be screened for autoimmune thyroid disease with a serum TSH level and for celiac disease with assay for tissue transglutaminase (IgA) antibodies.
A 62-year-old man with advanced renal disease due to diabetic nephropathy has an estimated glomerular filtration rate of 15 mL/min, serum calcium of 9.3 mg/dL, and phosphate of 5.2 mg/dL. Which of the following is most likely to be elevated in this patient? A. Calcitriol B. Ionized calcium C. Parathyroid hormone D. Serum iron E. TSH
Correct answer C Educational objective: Secondary hyperparathyroidism in chronic renal failure is characterized by hypo- or normocalcemia, hyperphosphatemia, and increased parathyroid hormone levels.
Urine is positive for microalbumin. Which of the following measures is most likely to achieve a hemoglobin A1c <6.0% in a year in this patient? A. Adding a sulfonylurea B. Adding an incretin mimetic C. Bariatric surgery D. Continued intensive nutritional counseling E. Starting insulin therapy
Correct answer C Educational objective: Severely obese patients with type 2 diabetes have a higher likelihood of achieving diabetic remission with bariatric surgery compared to conventional medical therapy.
A 66-year-old woman with a 10-year-history of type 2 diabetes mellitus comes to the physician because of unsteadiness. She stumbles occasionally and has had several near-fall episodes over the last 6 months. Her medications include aspirin, lisinopril, metformin, sitagliptin, and a statin. Her blood pressure is 133/70 mm Hg supine and 135/76 mm Hg while standing, and her pulse is 82/min. Her body mass index is 32 kg/m . Pinprick and vibration sensations are decreased in both feet. Ankle reflexes are absent bilaterally. Laboratory results are as follows: Hemoglobin 7.5% Vitamin B 150 pg/mL (200-800 pg/mL) Which of the following most likely contributed to this patient's current condition? A. Autonomic neuropathy B. Jejunal villous atrophy C. Metformin therapy D. Pancreatic atrophy E. Proteinuria
Correct answer C Educational objective: The risk of developing vitamin B deficiency is increased in patients treated with metformin. The risk increases with older patients, higher metformin doses, and longer durations of treatment. Metformin is thought to impair the terminal ileal absorption of vitamin B by inhibiting calcium-dependent binding of the intrinsic factor-B complex to its receptor.
A 42-year-old woman comes to the office for a preventive visit. She is concerned because a friend was recently diagnosed with breast cancer at age 45 and inquires whether screening mammography is recommended. The patient does not smoke, exercises regularly, eats a balanced diet, and has no significant cardiovascular risk factors. Cervical cancer screening with cytology and human papillomavirus testing a year ago was negative. The patient has no personal or family history of breast or gynecologic cancer. Vital signs and physical examination are normal. Which of the following is the most appropriate statement regarding breast cancer screening in this patient? A. If you choose to be screened for breast cancer, an MRI alone is a better choice than mammography. B. Mammography before age 50 does not lower the risk of death from breast cancer. C. Mammography in women under age 50 has a higher false-positive rate than in women over age 50. D. You should be tested for BRCA1 and BRCA2 before deciding whether to undergo mammography. E. You should perform monthly self breast examination, and we will order mammography if you find an abnormality.
Correct answer C Educational objective: Women at average risk for breast cancer should be screened with mammography every 2 years from age 50-75. Women age 40-49 have a lower incidence of breast cancer and a greater risk for false-positive mammography; screening in this population should be based on shared decision-making with the patient and physician.
A 16-year-old boy comes to the physician for a routine health maintenance visit concerned about breast enlargement. Over the past few months, he has noticed his left breast getting larger but has no pain. He smokes marijuana daily and occasionally snorts amphetamines at weekend parties. The patient is a B student and on the wrestling team at school. His mother has breast cancer, and his maternal aunt and grandmother had breast and ovarian cancers. He is at the 50th percentile for weight and height. Blood pressure is 115/68 mm Hg and pulse is 68/min. The patient has enlarged, tender, palpable left breast tissue extending about 1.5 cm beyond the areola with no abnormalities of the right chest. He has Tanner stage V male genitalia and copious axillary hair. No lymphadenopathy is palpated in the cervical, axillary, and groin regions. Testicular volume is 22 mL. Which of the following is the most likely cause of this patient's breast enlargement? A. Amphetamine abuse B. Breast malignancy C. Chromosomal abnormality D. Marijuana abuse E. Normal pubertal changes
Correct answer D Educational objective: Adolescents who abuse marijuana, alcohol, amphetamines, heroin, or methadone should be aware that gynecomastia may develop along with other serious psychosocial and medical problems. Treatment of drug-induced gynecomastia includes discontinuation of the offending drug.
A 34-year-old alcoholic man comes to the emergency department due to recurrent vomiting over the last week. The patient's blood pressure is 112/78 mm Hg when supine and 94/57 mm Hg when standing. Heart rate is 110/min with a regular rhythm. BMI is 18 kg/m . He looks disheveled, and his mucous membranes are dry. Abdominal examination reveals no tenderness to palpation. Initial laboratory results are as follows:Sodium 136 mEq/LPotassium 4.5 mEq/LChloride 103 mEq/LBicarbonate 21 mEq/LBlood ureanitrogen 27 mg/dLCreatinine 1.1 mg/dLGlucose 98 mg/dLCalcium 9.2 mg/dLMagnesium 1.5 mg/dLPhosphate 3.2 mg/dLCreatine kinase 90 U/L (39-238 U/L)Lipase 45 U/L (<95 U/L)Albumin 2.9 g/dLThe patient is treated with intravenous fluids, dextrose and thiamine solution, and folic acid supplementation. On the second day of hospitalization, he has shortness of breath, palpitations, and severe muscle weakness, and he can barely raise his arms or walk. Which of the following most likely accounts for this patient's current symptoms? A. Hypercalcemia B. Hypoglycemia C. Hyponatremia D. Hypophosphatemia E. Wernicke's encephalopathy
Correct answer D Educational objective: Chronic alcoholic patients are frequently phosphate depleted even though serum phosphate levels may be normal initially. Refeeding these patients may lead to shifts in phosphate intracellularly with a resulting decrease in serum phosphate. This may result in the development of rhabdomyolysis as many of these patients have an underlying myopathy.
An 11-year-old girl is brought to the emergency department with abdominal pain, vomiting, fatigue, polyuria, and polydipsia. Her temperature is 36.7 C (98 F), blood pressure is 95/60 mm Hg, pulse is 120/min, and respirations are 24/min. Physical examination shows dry skin and mucous membranes. The girl is tired-appearing but alert and oriented to person, place, and time. Laboratory results are as follows: Complete blood count Hemoglobin 13 g/dL Platelets 320,000/µL Leukocytes 14,000/µLSerum chemistry Sodium 136 mEq/L Potassium 5 mEq/L Chloride 100 mEq/L Blood urea nitrogen 26 mg/dL Creatinine 1 mg/dL Glucose 470 mg/dL Venous blood gas pH 7.15 Bicarbonate 8 mEq/L Urinalysis shows 3+ ketones and 3+ glucose. Which of the following is the most appropriate next step in management of this patient? A. Abdominal ultrasoundB. Head CT scanC. Intravenous insulinD. Intravenous normal saline bolusE. Urine culture
Correct answer D Educational objective: Diabetic ketoacidosis (DKA) is defined as metabolic acidosis (pH <7.3 or serum bicarbonate <15 mEq/L) in the setting of hyperglycemia (serum glucose >200 mg/dL). Treatment consists of volume repletion with a normal saline bolus given over 1 hour followed by initiation of an insulin drip and potassiumcontaining intravenous fluids.
An 18-year-old woman comes to the office for evaluation of vaginal discharge that is thin, gray, and malodorous. The patient has had no vulvovaginal pruritus or pelvic pain. She is sexually active with multiple partners and uses oral contraceptives but does not consistently use condoms. Her general health is good. She recently completed a course of antibiotics for uncomplicated cystitis. Review of systems is negative. Vital signs are normal. Speculum examination reveals a gray, adherent, malodorous discharge on the walls of the vaginal vault. The cervix appears normal and there is no cervical motion tenderness. Vaginal pH is 5. Microscopic examination is shown below: Which of the following is the best therapy for this patient? A. Ceftriaxone B. Doxycycline C. Fluconazole D. Metronidazole E. Trimethoprim-sulfamethoxazole
Correct answer D Educational objective: Diagnostic criteria for bacterial vaginosis include a thin, gray, malodorous vaginal discharge; a vaginal pH >4.5; an amine odor after the application of potassium hydroxide (eg, "whiff" test); and clue cells on wet mount microscopy. Treatment is with metronidazole or clindamycin.
A 22-year-old woman comes to the emergency department with sudden-onset, right-sided pelvic pain that started while exercising. The patient has no associated nausea, vomiting, or changes in bowel function. Her last menstrual period was 3 weeks ago and she is not on contraception. Vital signs are normal. The abdomen is slightly tender to palpation but there is no rebound or guarding. Pregnancy test is negative. Pelvic ultrasound shows a 4-cm, thin-walled ovarian cyst with Doppler flow and minimal free fluid in the posterior cul-de-sac. Hemoglobin is normal. Which of the following is the best next step in management of this patient? A. CA-125 level B. Diagnostic laparoscopy C. Empiric antibiotics D. Observation and reassurance E. Ovarian biopsy
Correct answer D Educational objective: Follicular ovarian cysts are typically smooth, thin-walled, and <10 cm. Rupture of these cysts can occur with vigorous activity, resulting in sudden onset of pelvic pain. Hemodynamically stable patients with a follicular ovarian cyst are managed with observation and reassurance.
A 40-year-old woman comes to the office due to diabetes mellitus. The patient has a history of type 2 diabetes mellitus treated with metformin for the past 3 years; home blood glucose levels are 90-130 mg/dL. She is adherent with her medication and recommended lifestyle measures and has lost significant weight since being diagnosed. Since she stopped her birth control medication 6 months ago, her menstrual periods have been unpredictable, prolonged, and heavy. Temperature is 37.2 C (99 F), blood pressure is 118/68 mm Hg, and pulse is 94/min. BMI is 28 kg/m . Physical examination is unremarkable, except for mucosal pallor. Laboratory results are as follows:Hemoglobin 9 g/dLMean corpuscular volume 65 µmPlatelets 320,000/mmLeukocytes 7,500/mmFerritin, serum 12 ng/mL(normal12-150)Hemoglobin A1c 7.6% (6 months ago: 6.8%)Vitamin B , serum 480 pg/mL (normal: 180-914)Iron supplementation is prescribed, and a referral to gynecology is made. In addition to continuing lifestyle changes, which of the following is this patient's best option for managing her diabetes mellitus? A. Add basal insulin B. Add glipizide C. Add glucagon-like peptide-1 analogue D. No change, repeat hemoglobin A1c in 3 months
Correct answer D Educational objective: Hemoglobin A1c (HbA1c) is vulnerable to changes in hemoglobin production and survival. Conditions associated with reduced red blood cell survival cause a misleadingly low HbA1c. Conversely, conditions associated with reduced red blood cell production (eg, iron deficiency) cause a misleadingly high HbA1c.
A 45-year-old man is concerned about developing diabetes. He has gained 6.8 kg (15 lb) over the last 5 years. The patient's medical history is unremarkable and he takes no medications. His father suffers from type 2 diabetes mellitus. BMI is 32 kg/m . Laboratory evaluation shows a fasting plasma glucose of 127 mg/dL and a hemoglobin A1c of 6%. Which of the following is the best management for this patient? A. Diagnose diabetes and start metformin B. Order an oral glucose tolerance test C. Prescribe a home glucose meter D. Repeat the fasting glucose E. Repeat the hemoglobin A1c
Correct answer D Educational objective: If the results from 2 different tests for diabetes mellitus are discordant, the test with a value above the diagnostic threshold should be repeated.
A 55-year-old-man comes to the office due to erectile dysfunction. He has normal libido but has been unable to maintain a satisfactory erection to have intercourse. The patient often wakes up in the morning with an erection. He has no history of testicular injury or peripheral vision loss. Medical history includes end-stage renal disease resulting from idiopathic focal segmental glomerulosclerosis. Peritoneal dialysis was started 6 months ago due to intractable fluid retention despite high-dose diuretic therapy, and he is being evaluated for possible renal transplant. The patient does not use tobacco, alcohol, or illicit drugs. Temperature is 37.1 C (98.7 F), blood pressure is 138/82 mm Hg, pulse is 76/min, and respirations are 12/min. BMI is 25 kg/m . Visual fields are normal on confrontation. Testes are normal in size and consistency, and cremasteric reflex is present. There is trace edema at the ankles and feet. Laboratory results are as follows: Hemoglobin 10.8 g/dL Creatinine 1.6 mg/dL Prolactin, serum 25 ng/mL Testosterone, serum 520 ng/dL (normal: 300-1,000) TSH 1.8 µU/mL Which of the following is the most appropriate next step in management of this patient's erectile dysfunction
Correct answer D Educational objective: Most men with end-stage renal disease experience sexual dysfunction (eg, low libido, erectile dysfunction, decreased frequency of intercourse, infertility). Contributing factors include vascular impairment, peripheral and autonomic neuropathy, gonadal dysfunction, psychological stress, concurrent medications, and medical comorbidities. Most patients respond to phosphodiesterase inhibitors (eg, sildenafil).
A 65-year-old woman comes to the physician for 5 weeks of persistent pain and mild jaw swelling after extraction of a left lower molar. She was diagnosed with multiple myeloma 8 months ago and treated with dexamethasone, lenalidomide, and zoledronic acid. Physical examination shows gingival ulceration at the site of the extraction with exposed bone and mild inflammation. Which of the following is the most likely diagnosis? A. Actinomycosis B. Gingival hyperplasia C. Necrotizing fascitis D. Osteonecrosis E. Plasmocytoma
Correct answer D Educational objective: Osteonecrosis of the jaw is a serious but uncommon complication of bisphosphonates or denosumab that occurs most often in patients with malignancy. Patients can develop pain, loosening of teeth, bony enlargement, gingival swelling or erythema, and exposed bone. Symptoms are more common at the site of dental extractions.
A 65-year-old man comes to the office a month after sustaining a nondisplaced left femoral neck fracture in a ground-level fall. He underwent internal fixation of the left hip and was discharged to a skilled nursing facility. The patient is now able to ambulate with a cane. He has no bone pain, previous fractures, diarrhea, or loss of height. Medical history is significant for hypertension treated with hydrochlorothiazide. The patient does not use tobacco or alcohol. There is no family history of osteoporosis or thyroid disorders. Since hospital discharge, the patient has taken adequate amounts of calcium and vitamin D via diet and oral supplementation. Vital signs are normal. The surgical scar has healed well and physical examination is unremarkable. Laboratory tests show normal complete blood count, basic metabolic panel, hepatic function tests, TSH, 24-hour urinary calcium, and 25-hydroxyvitamin D. DXA reveals T-scores of −2.0 in the femoral neck and −2.5 in the lumbar spine. Plain radiograph of the spine is normal. Which of the following is the most appropriate next step in evaluation of this patient?A. 24-hour urine cortisol measurementB. Serum prolactin levelC. Serum protein electr
Correct answer D Educational objective: Osteoporosis in men is often due to secondary causes. Elderly men with hip fractures frequently have hypogonadism and should be screened with an early-morning serum testosterone assay.
A 40-year-old woman comes to the physician for occasional low back pain after heavy exercise. Her blood pressure is 144/91 mm Hg and her pulse is 68/min. Her BMI is 29 kg/m . Physical examination is unremarkable. Laboratory results are as follows:Creatinine 1.5 mg/dL Calcium 11.8 mg/dL Albumin 4.0 g/dL TSH 2.7 µU/mL Repeat serum calcium is 11.5 mg/dL and parathyroid hormone (PTH) level is 89 pg/mL (normal 10-65 pg/mL). Dual energy x-ray absorptiometry (DEXA) reveals T-scores of -1.3 at the femoral neck, -0.9 at the lumbar spine, and -1.8 at the radius. Which of the following is the most appropriate next step in management of this patient? A. Bisphosphonate therapy B. Encourage hydration and follow-up in 6-12 months C. Parathyroid hormone related peptide (PTHrP) assay D. Referral for parathyroid surgery E. Renal ultrasound for nephrocalcinosis
Correct answer D Educational objective: Parathyroidectomy is indicated for patients with symptomatic primary hyperparathyroidism, as well as asymptomatic patients age <50, with renal insufficiency, low bone density with T-score <-2.5 (at any site), or serum calcium >1 mg/dL above upper limits of normal.
A 68-year-old man comes to the office to establish care. His medications include rosuvastatin for hypercholesterolemia and amlodipine for hypertension. He has gained 4.5 kg (10 lb) over the last 3 years since he quit smoking. His BMI is 31 kg/m . The patient's fasting blood glucose is 107 mg/dL and hemoglobin A is 5.9%. His fasting blood glucose was normal 3 years ago. Which of the following is the most appropriate intervention to prevent progression to overt type 2 diabetes mellitus in this patient? A. Changing from amlodipine to ramipril B. Discontinuing rosuvastatin C. Low-fat diet D. Metformin therapy E. Vitamin D supplementation
Correct answer D Educational objective: Patients with elevated fasting glucose and hemoglobin A levels are at increased risk for progression to overt diabetes mellitus. Interventions that can reduce the risk of diabetes include intensive lifestyle modification (diet, exercise, and weight loss) and metformin.
A 6-year-old girl is brought to the physician for a routine examination. She has no complaints and is enjoying first grade. Her favorite drink is orange juice and she enjoys chips and cookies. The girl has no medical problems and takes no medications. Review of all systems is negative. Several maternal relatives are obese, but no family members have heart disease or stroke. There are no smokers in the home. Body mass index was previously at the 80th percentile and is now at the 99th percentile. Vital signs are normal. Examination shows central adiposity. No acanthosis nigricans is seen. Which of the following is the best next step in management of this patient? A. 12-lead electrocardiogram B. Liver ultrasound C. No testing indicated at this age D. Serum lipid panel E. Serum thyroid function test
Correct answer D Educational objective: Pediatric obesity is increasingly common and is associated with early complications such as dyslipidemia, fatty liver disease, and hypertension. Fasting glucose, serum alanine aminotransferase, and lipid panel should be measured in these patients. Dietary and lifestyle modifications are the first-line treatment.
An 84-year-old woman comes to the office for evaluation of bleeding. The patient first noticed small amounts of blood on her underwear 2 weeks ago. She has been wearing pads that require changing once or twice a day with only a small amount of blood accumulating between changes. Her last Pap test at age 64 was normal. A small, hyperplastic rectal polyp was removed at age 80. Vital signs are normal. BMI is 38 kg/m . The speculum is difficult to insert; on examination there is mucosal pallor throughout the vagina with epithelial thinning and dryness. No mass or prolapse is visualized. Fecal occult blood test is negative. Which of the following is the best next step in management of this patient? A. Colonoscopy B. Colposcopy C. Reassurance and follow-up in 1-2 weeks D. Transvaginal ultrasound E. Vaginal estrogen therapy
Correct answer D Educational objective: Postmenopausal bleeding (PMB) requires assessment with either transvaginal ultrasound or endometrial biopsy to rule out endometrial cancer. Although vaginal atrophy is the most common cause of PMB, the incidence of endometrial cancer increases with age and should be ruled out.
A 14-year-old boy is brought to the office for a health maintenance visit. The patient is interviewed in private and says that he is concerned that he seems to be "growing breasts," but he has no pain or nipple discharge. He is not sexually active and does not use tobacco, alcohol, or illicit drugs. Weight is at the 75th percentile and height is at the 60th percentile. Blood pressure is 115/68 mm Hg and pulse is 68/min. Examination shows a 1.5-cm area of soft glandular tissue beneath both areolas. He has normal Tanner stage III male genitalia. Which of the following is the most appropriate next step in management of this patient? A. Obtain mammography B. Obtain serum β-hCG and testosterone levels C. Obtain testicular ultrasound D. Reassurance and discharge home E. Refer for surgical evaluation
Correct answer D Educational objective: Pubertal gynecomastia occurs in most adolescent boys, peaking at age 13-14 during Tanner stages III and VI. The breast tissue is usually <2 cm in diameter and resolves spontaneously within a year.
A 61-year-old woman comes to the physician for a preventive visit. She inquires about the need for bone density testing. The patient is an active smoker and has smoked 1/2 pack of cigarettes daily for the last 40 years. Which of the following is the strongest additional risk factor for osteoporosis in this patient? A. Diabetes mellitus B. High sodium intake C. Hyperlipidemia D. Low body weight E. Moderate alcohol intake
Correct answer D Educational objective: Screening for osteoporosis with dual-energy x-ray absorptiometry is recommended for all women age >65. Earlier screening can be considered for women with additional risk factors, including poor calcium/vitamin D intake, smoking, corticosteroid use, lack of weight-bearing exercise, low BMI, and heavy alcohol use.
A 70-year-old man with long-standing hypertension and chronic kidney disease has an estimated glomerular filtration rate of 19 mL/min/1.73m . His serum potassium level is 4.2 mEq/L, calcium is 9.8 mg/dL, and parathyroid hormone is 155 pg/mL (normal 10-65 pg/dL). Decreasing the intestinal absorption of which of the following would most likely benefit this patient? A. Calcium B. Iron C. Magnesium D. Phosphate E. Potassium
Correct answer D Educational objective: Secondary hyperparathyroidism in chronic kidney disease is characterized by renal phosphate retention, decreased calcium absorption, and elevated parathyroid hormone levels. Initial treatment includes ergocalciferol and lowering serum phosphate through dietary restriction and phosphate binders.
A 23-year-old woman comes to the office for a postpartum visit. The patient had a low-grade Pap test at her initial prenatal visit. She was followed for chronic hepatitis B infection, and her infant received hepatitis B vaccination and immunoglobulin after delivery. The patient's prenatal course and delivery were otherwise uncomplicated. She smoked a pack of cigarettes daily prior to pregnancy but cut back to 2 or 3 cigarettes a day and does not use alcohol or illicit drugs. The patient is breastfeeding her infant. She received all recommended vaccines during pregnancy but has not received the human papillomavirus (HPV) vaccination series. Which of the following is a contraindication to the HPV vaccine in this patient? A. Abnormal Pap test B. Breastfeeding status C. Hepatitis B infection D. No contraindication E. Tobacco use
Correct answer D Educational objective: The human papillomavirus (HPV) vaccination series is indicated for male and female patients to reduce the risk of HPV-associated disease (eg, anogenital and oropharyngeal cancer). Routine administration is between age 11 and 26, but vaccination can be given from age 9-45.
A 45-year-old man comes to the office for a follow-up visit. He was recently hospitalized for excessive urination and weight loss. His blood glucose level in the emergency department was 566 mg/dL with a normal anion gap. His hemoglobin A1c was 12.1%. The patient was not taking any medications before the hospitalization and had not seen a physician in more than 3 years. He was initially treated with an insulin infusion and subsequently transitioned to a basal-bolus regimen of insulins glargine and aspart. His blood glucose levels have been in the target range since discharge. The patient has a strong family history of type 2 diabetes. Examination is unremarkable except for BMI of 39 kg/m and neck acanthosis. The patient is very concerned about the possibility of lifelong insulin therapy. Which of the following is the most appropriate advice for this patient? A. Assay for anti-glutamic acid decarboxylase antibody will guide the need for continued insulin therapy B. He cannot use oral anti-diabetic medication and must continue lifelong insulin treatment C. He should consider going on an insulin pump D. Once his glucose is well controlled, he may be able to manage his diabetes without insul
Correct answer D Educational objective: Very high glucose levels can cause glucose toxicity, which is characterized by insulin resistance and impaired beta cell function. Short-term insulin therapy with tight glycemic control in patients with type 2 diabetes can reverse glucose toxicity and allow transition to oral agents.
A 34-year-old woman comes to the office due to darkening and thickening of the skin over her neck and groin areas. The patient otherwise feels well. Blood pressure is 130/80 mm Hg and pulse is 83/min. BMI is 31 kg/m . Skin examination findings are shown in the image. Similar lesions are found on the axilla and groin. The remainder of the physical examination is unremarkable.This patient's condition is most likely associated with which of the following? A. Addison disease B. Gastric adenocarcinoma C. Hemochromatosis D. Pellagra E. Polycystic ovary syndrome
Correct answer E Educational objective: Acanthosis nigricans is characterized by symmetric, hyperpigmented, velvety plaques in the axillae, groin, and neck. It is commonly associated with insulin resistance (eg, diabetes mellitus, polycystic ovary syndrome, obesity) in younger patients and, in rare cases, with gastrointestinal malignancy in older individuals.
A 2-year-old boy with Down syndrome is brought to the clinic for a routine health maintenance visit. His parents say that he has been constipated and has hard, pellet-like stools twice a week. His last bowel movement was 3 days ago, but he has had no abdominal pain or vomiting. The boy takes no medications and has a history of duodenal atresia successfully repaired shortly after birth. His vaccinations are up to date. Complete blood count and thyroid function testing were normal at his 1-year checkup. His growth chart shows declining height velocity from the 15th percentile to <5th percentile since birth. Weight is at the 50th percentile. Examination shows typical Down facies with a single palmar crease and protruding tongue. What is the best next step in management of this patient? A. Abdominal x-ray B. Complete blood count C. Fasting serum glucose D. Reassurance, follow-up in 1 month E. Serum thyroid-stimulating hormone
Correct answer E Educational objective: Annual screening for thyroid disorders should be performed in all patients with Down syndrome. Hypothyroidism in children commonly presents with declining growth velocity and should be evaluated by serum thyroid-stimulating hormone.
A 52-year-old woman comes to the office to follow up elevated blood pressure, which was 140/100 mm Hg during a routine preventive visit 4 weeks ago. The patient feels well and has experienced no recent weight changes, headaches, chest pain, palpitations, or polyuria. Medical history is notable only for gastroesophageal reflux disease managed with dietary modification, and she takes no medications. Screening colonoscopy at age 50 was normal. She drinks 1 or 2 glasses of wine most days and does not use tobacco or illicit drugs. The patient is an only child, and her parents have no unusual health history. Blood pressure is 138/100 mm Hg and pulse is 72/min. BMI is 27 kg/m . The patient appears mildly overweight but comfortable. Physical examination is otherwise unremarkable. Laboratory results are as follows: Serum chemistriesSodium 142 mEq/LPotassium 3.9 mEq/LCalcium 9.9 mg/dLCreatinine 1.1 mg/dLGlucose 95 mg/dL Lipid profileTotal cholesterol 205 mg/dLHDL 45 mg/dLLDL (calculated) 122 mg/dLTriglycerides 190 mg/dComplete blood countHemoglobin 13 g/dLLeukocytes 6,000/mmPlatelets 185,000/mm Urinalysis is normal. ECG shows normal sinus rhythm with no significant abnormalities. Which of the foll
Correct answer E Educational objective: Both hyperthyroidism and hypothyroidism can raise blood pressure. Untreated thyroid disease can impair the response to antihypertensive management, but many patients experience normalization of blood pressure when the underlying thyroid disorder is treated. Patients with hypertension warrant screening for thyroid disease in the initial laboratory evaluation.
A 26-year-old woman comes to the office for a health maintenance examination. Her aunt was recently diagnosed with stage 3 breast cancer and is currently undergoing treatment. The patient is worried about developing breast cancer and asks whether performing a monthly breast self-examination (BSE) would be beneficial. Which of the following is the most appropriate response regarding BSE? A. Patients who perform regular BSE have decreased breast cancer morbidity and mortality rates B. Performing regular BSE decreases the false-positive rate associated with breast cancer screening C. Regular monthly BSE decreases the rate of breast biopsy for suspicious lesions D. The ability to diagnose early-stage breast cancer improves in patients who perform regular BSE E. There are no proven benefits to routine BSE screening
Correct answer E Educational objective: Breast self-examination (BSE) is not recommended for breast cancer screening in any age group. BSE does not improve the rate of breast cancer diagnosis or breast cancer-associated death. In addition, BSE has a significant risk of false-positive results, leading to unnecessary diagnostic testing (eg, imaging, breast biopsy) for benign lesions.
A 55-year-old man comes to the office due to fatigue and inability to maintain satisfactory erections during intercourse. He has also noticed decreased early-morning erections and sexual desire. The patient has had no headaches, visual symptoms, polyuria, head injuries, or motor weakness. He has been trying to lose weight with diet and exercise. Medical history is notable for prediabetes, hypertension, and hypertriglyceridemia, and current medications include lisinopril, atorvastatin, and aspirin. The patient's father had type 2 diabetes mellitus and died of sudden cardiac arrest at age 74; his sister has hypothyroidism. He lives with his wife and 20-year-old son. Blood pressure is 142/74 mm Hg and pulse is 78/min. BMI is 38 kg/m . Body and facial hair distribution is normal. Visual fields are normal on confrontation. Testes are normal in size but have a soft consistency. The remainder of the physical examination is normal. Laboratory results are as follows: Hemoglobin 11 g/d Serum chemistry Sodium 136 mEq/LPotassium 3.8 mEq/L Chloride 90 mEq/L Bicarbonate 26 mEq/L Blood urea nitrogen 10 mg/dL Creatinine 1.2 mg/dLCalcium 9.2 mg/dLGlucose 104 mg/dLTSH 1.8 µU/mLHemoglobin A1c 5.9% Which of
Correct answer E Educational objective: Clinical features that warrant evaluation for androgen deficiency include decreased libido, erectile dysfunction, testicular atrophy, loss of body hair, gynecomastia, hot flashes, and an unexplained decrease in bone density. Nonspecific symptoms (eg, fatigue, weakness, weight changes) alone do not warrant testing in most cases. The initial step is a morning serum testosterone level, and low levels should be confirmed on repeat testing
A 63-year-old man without significant past medical history has decreased libido, decreased morning erections, and fatigue. His serum testosterone level on repeated testing is consistently low and he requests testosterone replacement therapy. Before initiating therapy, which of the following should be screened for in this patient? A. Coronary artery disease B. Deep venous thrombosis C. Hemochromatosis D. Obstructive lung disease E. Prostate cancer
Correct answer E Educational objective: Contraindications to testosterone replacement therapy include prostate or breast cancer, untreated severe sleep apnea, severe lower urinary tract symptoms, hematocrit >50%, uncontrolled heart failure, and elevated prostate-specific antigen levels. All patients should undergo prostate cancer screening prior to starting treatment.
A 24-year-old woman comes to the office due to excessive hair growth. She has had this problem for years and has tried different methods of hair removal. Vital signs are normal. BMI is 30 kg/m . Physical examination is normal other than an excess growth of dark, coarse hair on her upper lip, chin, upper abdomen, and buttocks. Which of the following is the most likely cause of this patient's condition? A. Congenital adrenal hyperplasia B. Familial hypertrichosis C. Idiopathic hirsutism D. Ovarian tumor E. Polycystic ovary syndrome
Correct answer E Educational objective: Hirsutism is excess terminal hair growth (dark, coarse hair) in androgen-dependent areas (eg, chin, upper lip, upper abdomen, back) in women. Polycystic ovary syndrome is the most common cause of true hirsutism and is associated with oligomenorrhea, obesity, type 2 diabetes, hypertension, and dyslipidemia.
A 42-year-old man comes to the office for further evaluation after a high blood pressure reading on an automated machine at a supermarket. He has no chronic medical problems and takes no medications. The patient has felt fatigued lately, which he attributes to stress at work. He also reports constipation and a 4.5-kg (10-lb) weight gain since he stopped exercising 9 months ago. The patient smoked cigarettes for a few years in college but has not smoked for the last 20 years. He does not use alcohol or illicit drugs. The patient lives with his girlfriend and has no children.Blood pressure is 145/95 mm Hg and pulse is 66/min. Repeat bloodpressure after 10 minutes is 135/85 mm Hg. BMI is 31 kg/m². The patient appears mildly obese, but the remainder of the examination is normal.Fasting laboratory results are as follows:Serum chemistries:Glucose: 95 mg/dLCreatinine: 1.1 mg/dLAspartate aminotransferase (SGOT): 44 U/LAlanine aminotransferase (SGPT): 53 U/LFasting lipid panel:Total cholesterol: 216 mg/dLHDL: 36 mg/dLLDL (calculated): 140 mg/dLTriglycerides: 200 mg/dLWhich of the following is the most appropriate next step in the management of this patient's metabolic abnormalities?A. Advise wei
Correct answer E Educational objective: Hypothyroidism is a common cause of dyslipidemia. Patients with dyslipidemia should be screened for hypothyroidism with serum TSH. Dyslipidemia due to hypothyroidism improves with levothyroxine therapy.
A 19-year-old woman is evaluated in the emergency department after an episode of loss of consciousness. The patient reports "feeling dizzy" prior to being found on the ground by her boyfriend, who witnessed her fall. Her boyfriend states "her eyes shut, and her arms and legs jerked for, maybe, 10 seconds or so." The patient has no medical conditions. Blood pressure is 95/65 mm Hg and pulse is 106/min. Physical examination shows an alert but anxious-appearing woman. Oropharyngeal examination shows no tongue lacerations. Laboratory findings are as follows. Sodium 131 mEq/L Potassium 2.4 mEq/L Chloride 82 mEq/L Bicarbonate 34 mEq/L Glucose 88 mg/dL Blood urea nitrogen 35 mg/dL Creatinine 1.1 mg/dL Arterial pH is 7.48. Urine chloride is low at 8 mEq/L. Which of the following is the most likely diagnosis in this patient? A. Active diuretic misuse B. Ecstasy intoxication C. Primary hyperaldosteronism D. Seizure disorder E. Surreptitious vomiting
Correct answer E Educational objective: In patients with metabolic alkalosis, urine chloride can differentiate saline-unresponsive (eg, primary hyperaldosteronism) from saline-responsive (eg, vomiting, recent diuretic use) causes. Urine chloride is low (<20 mEq/L) in saline-responsive cases.
A 27-year-old woman, gravida 1 para 0 aborta 1, comes to the office to discuss infertility issues. The patient and her husband have been having unprotected intercourse for 14 months. Her menstrual cycles are every 30 days, last 3-4 days, and are accompanied by breast tenderness. She had a spontaneous abortion at 7 weeks gestation with a different partner 5 years ago. The patient has no medical conditions or surgeries. Her 30-year-old husband has no medical conditions and takes no medications. Vital signs and physical examination are normal. Which of the following is the best next step in management of this patient? A. Antiphospholipid antibody testing B. Basal body temperature monitoring C. Hysteroscopy D. Laparoscopy E. Semen analysis
Correct answer E Educational objective: Infertility is the lack of conception after ≥12 months of unprotected intercourse in women age <35 (or ≥6 months in women age ≥35). Initial evaluation begins with semen analysis, assessments of ovarian reserve and ovulatory function, and evaluation of tubal patency
A 27-year-old athlete comes to the physician with a 2-month history of fatigue, erectile dysfunction, and muscle weakness. On examination, he has well-developed musculature but mild gynecomastia and small, soft testes. Which of the following laboratory abnormalities would confirm surreptitious androgen abuse in this patient? A. High prolactin levels B. High TSH levels C. Increased insulin-like growth factor-1 levels D. Increased transferrin saturation E. Low luteinizing hormone
Correct answer E Educational objective: Laboratory testing in patients with anabolic-androgenic steroid abuse shows increased hematocrit and decreased levels of endogenous testosterone, LH, and FSH.
A 44-year-old woman comes to the office for evaluation of a small breast lump that has been present for the past 3 months. The patient has no chronic medical conditions and does not use tobacco or alcohol. There is no family history of breast cancer. Vital signs are stable. BMI is 38 kg/m . Physical examination reveals no skin changes or dimpling. There are no palpable axillary or supraclavicular lymph nodes. The breasts are symmetric, diffusely cystic, and nontender. There is a firm 2-cm nodule at the 1 o'clock position on the left breast. A diagnostic mammogram shows dense breast tissue and no mammographic abnormalities. Which of the following is the best next step in management of this patient? A. Core needle biopsy B. MRI of the breast C. Repeat mammogram in 1 year D. Surgical excision of the mass E. Ultrasound of the breast
Correct answer E Educational objective: Mammography is first-line screening for breast mass evaluation in nonpregnant women age ≥30. Due to the risk of false-negative results, a normal mammogram requires further evaluation with ultrasound.
A 78-year-old woman, gravida 6 para 6, comes to the office due to vaginal spotting and malodorous vaginal discharge. She has felt a bulge protruding from the vagina for many years that is becoming more difficult to push up to relieve discomfort. The patient cannot always control her bladder and uses an incontinence pad. Her last cervical cancer screening was at age 65, and she has always had normal Pap tests. She has not been sexually active for over 30 years. Pelvic ultrasound reveals a small uterus with a homogeneous endometrium of 2 mm thickness. Which of the following is the best next step in management of this patient? A. Colposcopy B. Endometrial biopsy C. Gonorrhea and chlamydia nucleic acid amplification testing D. Hysteroscopy E. Urologic testing
Correct answer E Educational objective: Pelvic organ prolapse may present with pelvic pressure and a vaginal bulge that results in ulceration of the prolapsed tissue (eg, bleeding, discharge). Patients with pelvic organ prolapse often have concomitant urinary incontinence and require further evaluation with urologic testing.
A 45-year-old man is evaluated for hypertension. He has no symptoms except for mild fatigue. He does not use tobacco, alcohol, or illicit drugs. The patient's blood pressure is 160/94 mm Hg. Serum potassium is 3.2 mEq/L, creatinine is 0.9 mg/dL, and sodium is 146 mEq/L. Further workup shows a plasma aldosterone/plasma renin activity ratio of 45 (normal <20). CT of the abdomen shows a 2-cm, round right adrenal mass. He declines surgery for removal of the mass. Which of the following is the best initial therapy for this patient? A. Chlorthalidone B. Clonidine C. Hydralazine D. Ramipril E. Spironolactone
Correct answer E Educational objective: Primary hyperaldosteronism is usually due to an adrenal adenoma or bilateral adrenal hyperplasia. Surgery is preferred for unilateral adrenal adenoma. Medical therapy with aldosterone antagonists (eg, spironolactone, eplerenone) is recommended for patients with bilateral adrenal hyperplasia or with unilateral adrenal adenoma who either refuse surgery or are poor surgical candidates.
A 48-year-old overweight man with type 2 diabetes mellitus comes to the physician for an ulcer on his right foot. His vital signs are within normal limits. Examination shows a 1.3 x 2 cm ulcer on the plantar surface beneath the head of the fifth metatarsal. Lower extremity skin examination is otherwise normal. Which of the following is the best test to identify the primary underlying risk factor for this patient's ulcer? A. 6-minute walk test B. Ankle-brachial index C. Capillary refill time D. Knee reflex assessment E. Monofilament testing
Correct answer E Educational objective: Risk factors for diabetic foot ulcers include diabetic neuropathy, previous foot ulceration, vascular disease, and foot deformity. Diabetic neuropathy is the most important contributing factor and is found in >80% of patients with ulcers. Monofilament testing predicts the risk of future ulcers.
A 28-year-old woman comes to the office with a small lump in her neck. She has lost 2.3 kg (5 lb) over the last 6 months but has no other symptoms. The patient has no personal or family history of significant medical conditions. Serum TSH level is 0.01 µU/mL (normal: 0.5-5). Thyroid ultrasound reveals a 1.1-cm, round, hypoechoic nodule in the right lobe of the thyroid. There is no lymph node enlargement. Which of the following is the most appropriate course of action in this patient? A. Antithyroid peroxidase antibody assay B. Fine-needle aspiration C. Repeat thyroid ultrasound in 12 months D. Right thyroid lobectomy E. Thyroid scintigraphy
Correct answer E Educational objective: TSH measurement and thyroid ultrasound should be performed in all patients with thyroid nodules. Low/suppressed TSH suggests a hyperfunctioning nodule, which should be confirmed with thyroid scintigraphy. Hyperfunctioning nodules are usually benign.
A 50-year-old woman with newly diagnosed hypertension is started on chlorthalidone therapy. Thiazide diuretics are most likely to exacerbate which of the following disorders? A. Adrenal insufficiency B. Crohn disease C. Hypothyroidism D. Osteoporosis E. Primary hyperparathyroidism
Correct answer E Educational objective: Thiazide diuretics cause a decrease in urinary calcium excretion and can result in mild hypercalcemia. In patients with preexisting primary hyperparathyroidism, thiazide diuretics can result in additional elevation of serum calcium concentrations and "unmask" the hyperparathyroidism.
A 62-year-old woman with mild chronic upper back pain is found to have several old, healed vertebral fractures in the thoracic spine. Her past medical history includes recurrent deep vein thrombosis with pulmonary embolism and erosive esophagitis from severe gastroesophageal reflux disease. Dual-energy x-ray absorptiometry reveals osteoporosis with the lowest T-score of -3.1 at the right femur. Which of the following is the most appropriate treatment option for this patient? A. Calcitonin nasal spray daily B. Oral raloxifene daily C. Oral risedronate weekly D. Teriparatide subcutaneous injection daily E. Zoledronic acid intravenous infusion yearly
Correct answer E Educational objectives: Bisphosphonates reduce the risk of vertebral and non-vertebral fractures, including hip fractures; they remain first-line agents to reduce the risk of osteoporotic fractures. Intravenous zoledronic acid is recommended for patients who cannot take oral bisphosphonates.